Đến nội dung

Phạm Hữu Bảo Chung nội dung

Có 549 mục bởi Phạm Hữu Bảo Chung (Tìm giới hạn từ 19-04-2020)



Sắp theo                Sắp xếp  

#450991 1+$\frac{2}{3}\sqrt{x-x^{2}...

Đã gửi bởi Phạm Hữu Bảo Chung on 16-09-2013 - 19:13 trong Phương trình, Hệ phương trình Lượng giác

Bài 4.

Giải

Phương trình (1) của hệ tương đương:

$x(x + y)\left [ (y + z) + z\right ] = \dfrac{1}{8}$

$\Rightarrow -xz(z - x) = \dfrac{1}{8} \Leftrightarrow 8xz(x - z) = 1$

Ta có phương trình thứ hai của hệ tương đương:
$(x + y + z)^2 + xy + 2xz - z^2 = \dfrac{-3}{4}$

 

$\Leftrightarrow x(y + z) + xz - z^2 = \dfrac{-3}{4} \Rightarrow x^2 - xz + z^2 = \dfrac{3}{4}$

 

Đặt $x - z = a, xz = b$, ta có hệ: $\left\{\begin{matrix}8ab = 1\\a^2 + b = \dfrac{3}{4}\end{matrix}\right. \Leftrightarrow \left\{\begin{matrix}b = \dfrac{1}{8a}\\8a^3 - 6a = - 1 \, (1)\end{matrix}\right.$

Phương trình (1) có 3 nghiệm: $a = \cos{\dfrac{2\pi}{9}}; a = \cos{\dfrac{4\pi}{9}}$  và $\cos{\dfrac{8\pi}{9}}$

Tìm được a, suy ra b. Từ đó tìm được x, z và y. Vì dài quá nên ngại làm quá

 

 




#450983 1+$\frac{2}{3}\sqrt{x-x^{2}...

Đã gửi bởi Phạm Hữu Bảo Chung on 16-09-2013 - 18:44 trong Phương trình, Hệ phương trình Lượng giác

@NTHMyDream: Bạn có thể biến đổi như sau:

$y \left ( 1 - \dfrac{y^2 - 1}{2}\right ) = \dfrac{y^2 - 1}{\sqrt{2}}$

$\Leftrightarrow y^3 + \sqrt{2}y^2 - 3y - \sqrt{2} = 0$

$\Leftrightarrow (y - \sqrt{2})(y + \sqrt{2} - 1)(y + \sqrt{2} + 1) = 0$

Được rồi bạn nhỉ?

 




#450973 $2x+1+x\sqrt{x^{2}+2}+(x+1)\sqrt{x^...

Đã gửi bởi Phạm Hữu Bảo Chung on 16-09-2013 - 18:03 trong Đại số

Bài 2

Giải

ĐK: $\dfrac{-1}{3} \leq x \leq 6$

Phương trình ban đầu tương đương:
$\left (\sqrt{3x + 1} - 4 \right ) + \left (1 - \sqrt{6 - x} \right ) + 3x^2 - 14x - 5 = 0$

 

$\Leftrightarrow \dfrac{3(x - 5)}{\sqrt{3x + 1} + 4} + \dfrac{x - 5}{1 + \sqrt{6 - x}} + (x - 5)(3x + 1) = 0$

$\Leftrightarrow (x - 5)\left (\dfrac{3}{\sqrt{3x + 1} + 4} + \dfrac{1}{1 + \sqrt{6 - x}} + 3x + 1\right ) = 0$

Do $x \geq \dfrac{-1}{3}$ nên $\dfrac{3}{\sqrt{3x + 1} + 4} + \dfrac{1}{1 + \sqrt{6 - x}} + 3x + 1 > 0$

 

Vậy, phương trình có nghiệm duy nhất x = 5

 

 




#450968 [Bất phương trình chứa căn]$\frac{x-\sqrt{x}...

Đã gửi bởi Phạm Hữu Bảo Chung on 16-09-2013 - 17:57 trong Phương trình - hệ phương trình - bất phương trình

Bài 2

Giải

ĐK: $x \geq \dfrac{5}{3}$

Bất phương trình tương đương:
$2x\left (\sqrt{3x - 5} + \sqrt{4x - 3}\right ) < 5\left (\sqrt{2x + 9} - 3 \right )\left (\sqrt{2x + 9} + 3 \right )$

$\Leftrightarrow 2x\left (\sqrt{3x - 5} + \sqrt{4x - 3}\right ) < 10x$

$\Leftrightarrow \sqrt{3x - 5} + \sqrt{4x - 3} < 5$ (Do $x \geq \dfrac{5}{3}$)

$\Leftrightarrow (x - 3)\left ( \dfrac{3}{\sqrt{3x - 5} + 2} + \dfrac{4}{\sqrt{4x - 3} + 3}\right ) < 0$

$\Leftrightarrow x < 3$

Kết hợp với điều kiện, ta có: $\dfrac{5}{3} \leq x < 3$

 

 




#450964 [Bất phương trình chứa căn]$\frac{x-\sqrt{x}...

Đã gửi bởi Phạm Hữu Bảo Chung on 16-09-2013 - 17:35 trong Phương trình - hệ phương trình - bất phương trình

Bài 3.

Giải

ĐK: $x \geq \dfrac{2}{3}$

Bất phương trình tương đương:

$(x^2 - x - 2) + \left (\sqrt{x + 2} - \sqrt{3x - 2} \right ) \leq 0$

$\Leftrightarrow (x - 2)(x + 1) + \dfrac{4 - 2x}{\sqrt{x + 2} + \sqrt{3x - 2}} \leq 0$

$\Leftrightarrow (x - 2)\left [ x + 1 - \dfrac{2}{\sqrt{x + 2} + \sqrt{3x - 2}}\right ] \leq 0$

Ta thấy, với $x \geq \dfrac{2}{3}$ thì $x + 1 - \dfrac{2}{\sqrt{x + 2} + \sqrt{3x - 2}} \geq \dfrac{10 - 3\sqrt{6}}{6} > 0$

Vậy, bất phương trình ban đầu tương đương: $x \leq 2$
Kết hợp điều kiện, ta có: $\dfrac{2}{3} \leq x \leq 2$

 

 




#450962 [Bất phương trình chứa căn]$\frac{x-\sqrt{x}...

Đã gửi bởi Phạm Hữu Bảo Chung on 16-09-2013 - 17:24 trong Phương trình - hệ phương trình - bất phương trình

Bài 1.

Giải

ĐK: $x \geq 0$

Nhận thấy: $\sqrt{2(x^2 - x + 1)} = \sqrt{2\left ( x - \dfrac{1}{2}\right )^2 + \dfrac{3}{2}} > 1$

Vậy: $1 - \sqrt{2(x^2 - x + 1)} < 0$

Bất phương trình tương đương:

$x - \sqrt{x} \leq 1 - \sqrt{2(x^2 - x + 1)}$
Do x = 0 không phải là nghiệm nên chia hai vế cho $\sqrt{x}$, ta được:
$\sqrt{x} - 1 - \dfrac{1}{\sqrt{x}} + \sqrt{2\left ( x + \dfrac{1}{x} - 1\right )} \leq 0$

 

Đặt: $\sqrt{x} - \dfrac{1}{\sqrt{x}} = t \Rightarrow t^2 + 2 = x + \dfrac{1}{x}$, ta được:
$t - 1 + \sqrt{2t^2 + 2} \leq 0$

Bạn tự giải tiếp nhé :)




#450934 $S=(x+y)^{2}-\sqrt{9-x-y}+\frac{1...

Đã gửi bởi Phạm Hữu Bảo Chung on 16-09-2013 - 14:14 trong Bất đẳng thức và cực trị

Giải

Theo giả thiết, ta có: $x + y - 1 \geq 0 \Leftrightarrow x + y \geq 1$

Mặt khác, ta có:

$x + y - 1 = \sqrt{2x - 4} + \sqrt{y + 1} = \sqrt{2(x - 2)} + \sqrt{1(y + 1)}$

 

$\leq \dfrac{x}{2} + \dfrac{y + 2}{2} = \dfrac{x + y + 2}{2} \Leftrightarrow x + y \leq 4$

 

Đặt $x + y = t$ với $t \in [1; 4]$, ta có:
$P = f(t) = t^2 - \sqrt{9 - t} + \dfrac{1}{\sqrt{t}}$

Có $f’(t) = 2t + \dfrac{1}{2\sqrt{9 - t}} - \dfrac{1}{2t\sqrt{t}}$

Với $t \in [1; 4]$ thì $f’(t) > 0$

Vậy, hàm đồng biến trên [1; 4]. Suy ra: $f(1) \leq f(x) \leq f(4)$

 

Khi đó:
$Min_P = 2 - 2\sqrt{2}$ khi $x = 2; y = -1$

$Max_P = \dfrac{33 - 2\sqrt{5}}{2}$ khi $x = 4; y = 0$

 

 




#450744 $\frac{1}{cos^{2}x}-\frac{1...

Đã gửi bởi Phạm Hữu Bảo Chung on 15-09-2013 - 17:06 trong Phương trình, Hệ phương trình Lượng giác

Giải

ĐK: $x \neq \dfrac{k\pi}{2}$, $x \neq \dfrac{-\pi}{3} + k\pi$ và $x \neq \dfrac{\pi}{6} + k\pi \, (k \in Z)$

Nhận xét: $\cot{\left ( \dfrac{\pi}{6} - x\right )} = \tan{\left [ \dfrac{\pi}{2} - \left ( \dfrac{\pi}{6} - x \right )\right ]} = \tan{\left ( x + \dfrac{\pi}{3}\right )}$

Vậy: $\cot{\left ( \dfrac{\pi}{6} - x\right )}\cot{\left ( x + \dfrac{\pi}{3}\right )} = 1$

Vậy, phương trình tương đương:
$\dfrac{\sin^2{x} - \cos^2{x}}{\sin^2{x}\cos^2{x}} = \dfrac{8}{3}$

$\Leftrightarrow -\cos{2x} = \dfrac{2}{3}\sin^2{2x}$

$\Leftrightarrow \dfrac{2}{3}\cos^2{2x} - \cos{2x} - \dfrac{2}{3} = 0 $

Còn lại bạn tự làm hén. Chú ý đối chiếu điều kiện.

 




#450738 $\sqrt{7x+y}-\sqrt{2x+y}=4$

Đã gửi bởi Phạm Hữu Bảo Chung on 15-09-2013 - 16:45 trong Phương trình - hệ phương trình - bất phương trình

Giải

ĐK: $x \geq \dfrac{-8}{5}$, $2x + y \geq 0$ và $7x + y \geq 0$

Phương trình (1) của hệ tương đương:

$\sqrt{7x + y} = \sqrt{2x + y} + 4$
$\Leftrightarrow 7x + y = 2x + y + 16 + 8\sqrt{2x + y} \Rightarrow 2\sqrt{2x + y} = \dfrac{5}{4}x - 4$

Thế vào phương trình (2), ta được: $\dfrac{5}{4}x - 4 - \sqrt{5x + 8} = 2$

$\Leftrightarrow \sqrt{5x + 8} = \dfrac{5}{4}x - 6 \Leftrightarrow  \left\{\begin{matrix}x \geq \dfrac{12}{5}\\5x + 8 = \left ( \dfrac{5}{4}x - 6\right )^2 \end{matrix}\right.$

$\Rightarrow x = \dfrac{56}{5} \Rightarrow y = …$

                                




#450733 $\left\{\begin{matrix} (\frac{1-...

Đã gửi bởi Phạm Hữu Bảo Chung on 15-09-2013 - 16:33 trong Phương trình - hệ phương trình - bất phương trình

Giải

ĐK: $x \neq 0$

Phương trình thứ hai của hệ tương đương:
$(xy + 2)^2 - 2\dfrac{1}{x}(xy + 2) + \dfrac{1}{x^2} = 0$

$\Leftrightarrow \left (xy + 2 - \dfrac{1}{x} \right )^2 = 0 \Rightarrow xy + 2 = \dfrac{1}{x}$

 

Thế vào phương trình thứ nhất của hệ, ta có:
$\left ( \dfrac{1}{x^2} - 1 \right )^3 + \dfrac{1}{x} - \dfrac{1}{2} = \left ( \dfrac{1}{x^2} - \dfrac{2}{x}\right )^3$

 

Đặt $a = \dfrac{1}{x} \Rightarrow  (a^2 - 1)^3 + a - \dfrac{1}{2} = (a^2 - 2a)^3$


$\Leftrightarrow \left [(a^2 - 1)^3 - (a^2 - 2a)^3 \right ] + \dfrac{2a - 1}{2} = 0$

$\Leftrightarrow (2a - 1)\left [ (a^2 - 1)^2 + (a^2 - 1)(a^2 - 2a) + (a^2 - 2a)^2 + \dfrac{1}{2}\right ] = 0$

$\Rightarrow a = \dfrac{1}{2} \Rightarrow x = 2 \Rightarrow y = \dfrac{-3}{4}$

 




#450727 Tìm thiết diện của tứ diện ABCD

Đã gửi bởi Phạm Hữu Bảo Chung on 15-09-2013 - 16:15 trong Hình học không gian

Giải

Trường hợp 1. BO // CD

Qua M, dựng MK song song với BO . Suy ra: MK //CD.

Suy ra: K $\in$ (MCD)

Vậy: CDK chính là thiết diện của (CDM) với hình chóp.

 

Trường hợp 2. BO cắt CD

Trên mặt phẳng (BCD)¸ gọi H = BO $\cap$ CD

Trên mặt phẳng (BAH), kéo dài HM cắt AB tại K. (K thuộc đoạn thẳng AB)

Nối K với C và D, tam giác CDK chính là thiết diện tạo bởi (CDM) với hình chóp.

Thật vậy:

Vì H $\in$ CD nên H $\in$ (CDM) mà M $\in$ (CDM)

Vì vậy K = MH $\cap$ AB $\in$ (CDM)

Do đó: CKD chính là thiết diện cần tìm.

Chú ý rằng: Ở cả 2 TH, nếu K nằm ngoài đoạn AB thì (CDM) có thiết diện với hình chóp là đoạn CD

 




#450705 Viết phương trình tiếp tuyến với $(C):y=\frac{x^4}{4...

Đã gửi bởi Phạm Hữu Bảo Chung on 15-09-2013 - 15:48 trong Hàm số - Đạo hàm

Giải

Vì đường thẳng $x = m$ không phải là tiếp tuyến của (C) nên:

Đường thẳng cần tìm có dạng là d: $y = kx + m$.

Gọi A$(x_o; y_o)$ là tọa độ tiếp điểm. Khi đó, (d) là tiếp tuyến của (C) khi:
$\left\{\begin{matrix}\dfrac{x_o^4}{4} + \dfrac{x_o^2}{2} + 2 = kx_o + m\\x_o^3 + x_o = k\end{matrix}\right. \Leftrightarrow \left\{\begin{matrix}m = - \dfrac{3}{4}x_o^4 - \dfrac{1}{2}x_o^2 + 2 \\ k = x_o^3 + x_o\end{matrix}\right. $

                                                                     

Theo giả thiết: $d_{(A; d)} = \dfrac{9}{4\sqrt{5}} \Leftrightarrow \dfrac{|m - 3|}{\sqrt{k^2 + 1}} = \dfrac{9}{4\sqrt{5}}$

$\Leftrightarrow \dfrac{\left | - \dfrac{3}{4}x_o^4 - \dfrac{1}{2}x_o^2 + 2\right |}{\sqrt{x_o^2(x_o^2 + 1)^2 + 1}} = \dfrac{9}{4\sqrt{5}}$

$\Leftrightarrow \dfrac{3x_o^4 + 2x_o^2 + 4}{\sqrt{x_o^2(x_o^2 + 1)^2 + 1}} = \dfrac{9}{\sqrt{5}}$

 

Đặt $x_o^2 = t \geq 0$, quy đồng và bình phương hai vế, ta có:

$5(3t^2 + 2t + 4)^2 = 1 + 81t(t + 1)^2$
 

$\Leftrightarrow 45t^4 - 21t^3 - 22t^2 - t - 1 = 0$

 

$\Leftrightarrow (t - 1)(45t^3 + 24t^2 + 2t + 1) \Leftrightarrow t = 1$
 

$\Rightarrow x = \pm 1 \Rightarrow \left[\begin{matrix}(d): y = 2x + \dfrac{3}{4}\\(d): y = - 2x + \dfrac{3}{4}\end{matrix}\right.$

 




#450442 giải phương trình

Đã gửi bởi Phạm Hữu Bảo Chung on 14-09-2013 - 23:37 trong Phương trình - hệ phương trình - bất phương trình

Biểu thức sau cùng hình như là $\sqrt{x^3 + (\sqrt{3} - 1)x^2 + 2x - 2 + 2\sqrt{3} }$ phải không nhỉ?

Chỗ $(\sqrt{3} - 1)x$ của bạn đấy? Nếu không đoạn đó có thể rút gọn mà.

Giải

Điều kiện xác định là:
$\left\{\begin{matrix}x^2 - 2x - 2 \geq 0\\1 - \sqrt{3} - x \geq 0 \\x^3 + (\sqrt{3} - 1)x^2 + 2x - 2 + 2\sqrt{3} \geq 0\end{matrix}\right.$

$\Leftrightarrow \left\{\begin{matrix}\left[\begin{matrix}x \geq 1 + \sqrt{3}\\x \leq 1 - \sqrt{3}\end{matrix}\right.\\x \leq 1 - \sqrt{3}\\(x^2 + 2)(x + \sqrt{3} - 1) \geq 0\end{matrix}\right.$

 

$\Leftrightarrow \left\{\begin{matrix}x \leq 1 - \sqrt{3}\\x \geq 1 - \sqrt{3}\end{matrix}\right. \Rightarrow x = 1 - \sqrt{3}$

 

Thử lại thấy không thỏa mãn. Vì vậy, phương trình này vô nghiệm.

 




#450436 $3\left ( x+y+z \right )+xyz\geq 10$

Đã gửi bởi Phạm Hữu Bảo Chung on 14-09-2013 - 23:22 trong Bất đẳng thức và cực trị

Giải

Đặt $x + y + z = p; xy + yz + zx = q$ và $xyz = r$.

Theo giả thiết: $q = 3 \Rightarrow r \leq \sqrt{\dfrac{q^3}{27}} = 1$

Ta có:
$(x^3 + y^3 + z^3)(x + y + z) \geq (x^2 + y^2 + z^2)^2$

 

$\Rightarrow (p^3 - 3pq + 3r)p \geq (p^2 - 2q)^2 \Leftrightarrow p^2q + 3pr \geq 4q^2$

 

$\Leftrightarrow 3p^2 + 3pr - 36 \geq 0 \Leftrightarrow p^2 + pr - 12 \geq 0 \, (1)$

Vì $x, y, z > 0$ nên $p, r > 0$. Vì vậy, từ (1), suy ra: $p \geq \dfrac{\sqrt{r^2 + 48} - r}{2}$

 

Vậy, ta cần chứng minh: $\dfrac{\sqrt{r^2 + 48} - r}{2} \geq \dfrac{10 - r}{3} \Leftrightarrow 3\sqrt{r^2 + 48} \geq r + 20$

$\Leftrightarrow r^2 - 5r + 4 \geq 0 \Leftrightarrow (r - 1)(r - 4) \geq 0$

BĐT trên đúng với $r \leq 1$. Vậy, ta có điều phải chứng minh.

 

 




#450415 $\left\{\begin{matrix} x^2y^3+3x^2-4x+2=0...

Đã gửi bởi Phạm Hữu Bảo Chung on 14-09-2013 - 22:53 trong Phương trình - hệ phương trình - bất phương trình

Bài 4

Giải

Vì  x = 0 không phải nghiệm của hệ nên hệ ban đầu tương đương:
$\left\{\begin{matrix}3y + 55 = \dfrac{64}{x^3}\\x(y + 1)^3 = 12 + 52x \end{matrix}\right. \Leftrightarrow \left\{\begin{matrix}\dfrac{64}{x^3} - 3(y + 1) = 52\\(y + 1)^3 - \dfrac{12}{x} = 52\end{matrix}\right.$
Đặt $\left\{\begin{matrix}\dfrac{4}{x} = a\\y + 1 = b\end{matrix}\right.$, ta được:
$\left\{\begin{matrix}a^3 - 3b = 52\\b^3 - 3a = 52\end{matrix}\right. \Leftrightarrow \left\{\begin{matrix}a^3 - b^3 + 3(a - b) = 0\\a^3 - 3b= 52\end{matrix}\right.$

$\Leftrightarrow \left\{\begin{matrix}(a - b)(a^2 + ab + b^2 + 3) = 0\\a^3 - 3b = 52\end{matrix}\right. \Leftrightarrow \left\{\begin{matrix}\left[\begin{matrix}a = b\\a^2 + ab + b^2 + 3 = 0 \,(VN)\end{matrix}\right.\\a^3 - 3b = 52\end{matrix}\right.$

$\Leftrightarrow \left\{\begin{matrix}a = b\\a^3 - 3a - 52 = 0\end{matrix}\right. \Leftrightarrow \left\{\begin{matrix}a = b\\(a - 4)(a^2 + 4a + 13) = 0\end{matrix}\right. $

 

$\Leftrightarrow a = b = 4 \Rightarrow \left\{\begin{matrix}x = 1\\y = 3\end{matrix}\right.$

 

 




#450402 $\left\{\begin{matrix} x^2y^3+3x^2-4x+2=0...

Đã gửi bởi Phạm Hữu Bảo Chung on 14-09-2013 - 22:41 trong Phương trình - hệ phương trình - bất phương trình

Bài 3

Giải

Hệ phương trình ban đầu tương đương:
$\left\{\begin{matrix}y^2= \dfrac{8x}{x^2 + 1}\\y^3 + 8 + 2(x - 1)^2 = 0\end{matrix}\right.$

Từ phương trình thứ nhất của hệ, suy ra: $y^2 \leq 4 \Leftrightarrow -2 \leq y \leq 2$

Từ phương trình thứ hai của hệ, ta có: $y^3 + 8 = -2(x - 1)^2 \leq 0 \Rightarrow y \leq -2$

Vậy: $y = - 2 \Rightarrow x = 1$

 

 




#450290 $\left\{\begin{matrix} x^3+4y-y^3-16x=0...

Đã gửi bởi Phạm Hữu Bảo Chung on 14-09-2013 - 20:24 trong Phương trình - hệ phương trình - bất phương trình

Giải

Hệ ban đầu tương đương:
$\left\{\begin{matrix}x^3 - y^3 = 16x - 4y\\y^2 - 5x^2 = 4\end{matrix}\right. $

 

$\Rightarrow 4(x^3 - y^3) = (16x - 4y)(y^2 - 5x^2)$

$\Leftrightarrow 21x^3 - 4xy^2 - 5x^2y = 0 \Leftrightarrow x(21x^2 - 5xy - 4y^2) = 0$

 

Mới giải 1 bài y nguyên thế này :)))

 

 

 




#450277 Đề thi học sinh giỏi trường Chuyên Nguyễn Tất Thành

Đã gửi bởi Phạm Hữu Bảo Chung on 14-09-2013 - 19:59 trong Thi HSG cấp Tỉnh, Thành phố. Olympic 30-4. Đề thi và kiểm tra đội tuyển các cấp.

Giải

Hệ ban đầu tương đương:
$\left\{\begin{matrix}x^3 - 8y^3 = 6x - 10y\\x^2 + 8y^2 = 6\end{matrix}\right. \Rightarrow 6(x^3 - 8y^3) = (6x - 10y)(x^2 + 8y^2)$

$\Leftrightarrow 16y^3 - 24xy^2 + 5x^2y = 0 \Leftrightarrow y(16y^2 - 24xy + 5x^2 ) = 0$




#449550 Tìm giá trị nhỏ nhất và lớn nhất của $P=\frac{2\left(x^2+...

Đã gửi bởi Phạm Hữu Bảo Chung on 12-09-2013 - 00:48 trong Bất đẳng thức và cực trị

Giải

Nếu $y = 0 \Rightarrow P = 2$

Với $y \neq 0$. Đặt $t = \dfrac{x}{y}$. Ta có:

$P = \dfrac{2x^2 + 12xy}{1 + 2xy + y^2} = \dfrac{2x^2 + 12xy}{x^2 + 2y^2 + 2xy} = \dfrac{2t^2 + 12t}{t^2 + 2t + 2}$

Xét hàm số: $f(t) = \dfrac{2t^2 + 12t}{t^2 + 2t + 2}$

Có $f’(t) = \dfrac{-8t^2 + 8t + 24}{(t^2 + 2t + 2)^2}$; $f’(t) = 0 \Leftrightarrow t = \dfrac{1 \pm \sqrt{13}}{2}$

Lập bảng biến thiên, ta tìm được:
$P_{max} = 2\sqrt{13} - 4$ và $P_{min} = -2\sqrt{13} - 4$

Bạn tự tìm giá trị của x, y tương ứng nhé.

 

 

 




#449548 Giải phương trình $\left(8\cos^3x+1\right)^3=162\cos...

Đã gửi bởi Phạm Hữu Bảo Chung on 12-09-2013 - 00:27 trong Phương trình, Hệ phương trình Lượng giác

Giải

Đặt $\cos{x} = t \, (-1 \leq t \leq 1)$, phương trình trở thành:
$(8t^3 + 1)^3 = 27(6t - 1) \Leftrightarrow 8t^3 + 1 = 3\sqrt[3]{6t - 1} \, (1)$

Đặt $\sqrt[3]{6t - 1} = 2u$, kết hợp với (1), ta có hệ:
$\left\{\begin{matrix}8u^3 + 1 = 6t\\8t^3 + 1 = 6u\end{matrix}\right. \Leftrightarrow \left\{\begin{matrix}8(u^3 - t^3) = 6(t - u)\\8t^3 + 1 = 6u\end{matrix}\right.$

$\Leftrightarrow \left\{\begin{matrix}2(u - t)\left [ 4(u^2 + ut + t^2) + 3\right ] = 0\\8t^3 + 1 = 6u\end{matrix}\right. \Leftrightarrow \left\{\begin{matrix}\left[\begin{matrix}u = t\\4(u^2 + ut + t^2) + 3 = 0 \, (VN)\end{matrix}\right.\\8t^3 + 1 = 6u\end{matrix}\right.$

$\Leftrightarrow \left\{\begin{matrix}u = t\\8t^3 + 1 = 6t\end{matrix}\right. \Rightarrow 8\cos^3{x} - 6\cos{x} = -1 \Leftrightarrow \cos{3x} = \dfrac{-1}{2}$

Còn lại bạn tự làm nhé.

 

 




#449546 $\left\{\begin{matrix} (2012-3x)\sqrt...

Đã gửi bởi Phạm Hữu Bảo Chung on 12-09-2013 - 00:12 trong Phương trình - hệ phương trình - bất phương trình

Giải

ĐK: $x \leq 4; y \leq \dfrac{3}{2}; x \geq \dfrac{8y}{7}$ và $x \geq \dfrac{9y}{7}$

Đặt $\sqrt{4 - x} = a; \sqrt{3 – 2y} = b \, (a, b \geq 0)$

Phương trình (1) của hệ tương đương:
$(2000 + 3a^2)a - (2000 + 3b^2)b = 0 \Leftrightarrow 3(a^3 - b^3) + 2000(a - b) = 0$

$\Leftrightarrow (a - b)\left [3(a^2 + ab + b^2) + 2000\right ] = 0 \Leftrightarrow a = b \Rightarrow 2y = x - 1$

 

Thế $2y = x - 1$ vào phương trình (2) của hệ, ta được:
$$2\sqrt{3x + 4} + 3\sqrt{5x + 9} = x^2 + 6x + 13$$

Với điều kiện $x \geq \dfrac{-4}{3}$, phương trình trên tương đương:

$x^2 + x + 2\left (x + 2 - \sqrt{3x + 4}\right ) + 3\left (x + 3 - \sqrt{5x + 9}\right ) = 0$

$\Leftrightarrow x^2 + x + \dfrac{2(x^2 + x)}{x + 2 + \sqrt{3x + 4}} + \dfrac{3(x^2 + x)}{x + 3 + \sqrt{5x + 9}} = 0$

 

$\Leftrightarrow (x^2 + x)\left ( 1 + \dfrac{2}{x + 2 + \sqrt{3x + 4}} + \dfrac{3}{x + 3 + \sqrt{5x + 9}}\right ) = 0$

Do $x \geq \dfrac{-4}{3} \Rightarrow 1 + \dfrac{2}{x + 2 + \sqrt{3x + 4}} + \dfrac{3}{x + 3 + \sqrt{5x + 9}} > 0$

 

Vậy $x^2 + x = 0 \Rightarrow \left[\begin{matrix}x = 0 \Rightarrow y = \dfrac{-1}{2}\\x = -1 \Rightarrow y = -1\end{matrix}\right.$

 

 

 




#449491 Đề chọn đội tuyển

Đã gửi bởi Phạm Hữu Bảo Chung on 11-09-2013 - 22:04 trong Các dạng toán THPT khác

Bài 3. Ta đặt $\widehat{ACB} = \alpha$ cho dễ nhìn bạn nhé.

Giải

Gọi độ dài các cạnh BC, AC, AB lần lượt là a, b, c.

Khi đó, trong tam giác ABC, ta luôn chứng minh được:

$$m = \dfrac{2ab\cos{\dfrac{\alpha}{2}}}{a + b}$$
Bạn có thể tham khảo một số cách chứng minh trong nhiều tài liệu trên mạng :D

Vì vậy:

$m^2\tan{\dfrac{\alpha}{2}} = \dfrac{4a^2b^2\cos^2{\dfrac{\alpha}{2}}}{(a + b)^2}.\tan{\dfrac{\alpha}{2}} $

$= \dfrac{1}{2}.\dfrac{4ab}{(a + b)^2}.ab.2\sin{\dfrac{\alpha}{2}}\cos{\dfrac{\alpha}{2}} \leq \dfrac{1}{2}ab\sin{\alpha} = S$

 

Vậy, ta có điều phải chứng minh.

 

 




#449477 $x^{3}+\frac{x^{3}}{(x-1)^{...

Đã gửi bởi Phạm Hữu Bảo Chung on 11-09-2013 - 21:27 trong Phương trình, hệ phương trình và bất phương trình

Bài 2

Giải

ĐK: $x \neq 1$

Phương trình đã cho tương đương:
$$x^3 + \dfrac{x^3}{(x - 1)^3} + \dfrac{3x^2}{x - 1} - 2 = 0$$

Đặt $t = x + \dfrac{x}{x - 1} = \dfrac{x^2}{x - 1}$

$\Rightarrow t^3 = x^3 + \dfrac{x^3}{(x - 1)^3} + 3\dfrac{x^2}{x - 1 }.t \Rightarrow x^3 + \dfrac{x^3}{(x - 1)^3} = t^3 – 3t^2$

 

Khi đó, phương trình trở thành: $t^3 - 3t^2 + 3t - 2 = 0 \Leftrightarrow (t - 1)^3 = 1 \Leftrightarrow t = 2$

Phương trình ban đầu vô nghiệm.




#449475 $x^{3}+\frac{x^{3}}{(x-1)^{...

Đã gửi bởi Phạm Hữu Bảo Chung on 11-09-2013 - 21:19 trong Phương trình, hệ phương trình và bất phương trình

Bài 1 có thể biến đổi như sau:

Giải

Phương trình ban đầu tương đương:

$x^2 + \dfrac{(9x)^2}{(x + 9)^2} = 40$

$\Leftrightarrow \left ( x - \dfrac{9x}{x + 9}\right )^2 + 18\dfrac{x^2}{x + 9} = 40$

$\Leftrightarrow \dfrac{x^4}{(x + 9)^2} + 18\dfrac{x^2}{x + 9} - 40 = 0$

 

Đặt $t = \dfrac{x^2}{x + 9}$, ta được: $a^2 + 18a - 40 = 0$
 

 

 




#449470 $\frac{a}{ab+3c}$ $+$ $...

Đã gửi bởi Phạm Hữu Bảo Chung on 11-09-2013 - 21:09 trong Bất đẳng thức - Cực trị

Giải

Ta có:
$\dfrac{a}{ab + 3c} = \dfrac{a}{ab + (a + b + c)c} = \dfrac{a}{(a + c)(b + c)}$

Vậy:
$VT = \dfrac{a}{(a + c)(b + c)} + \dfrac{b}{(a + c)(a + b)} + \dfrac{c}{(a + b)(b + c)}$

$= \dfrac{a^2 + b^2 + c^2 + ab + bc + ca}{(a + b)(b + c)(c + a)} = \dfrac{(a + b + c)^2 - ab - bc - ca}{(a + b)(b + c)(c + a)}$

 

$\geq \dfrac{(a + b + c)^2 - \dfrac{1}{3}(a + b + c)^2}{\dfrac{8(a + b + c)^3}{27}} = \dfrac{9}{4(a + b + c)} = \dfrac{3}{4}$